Question about an inequality described by matrices












4














Let $A=(a_{ij})_{1 le i, j le n}$ be a matrix such that $sum_limits{i=1}^{n} a_{ij}=1$ for every $j$, and $sum_limits{j=1}^n a_{ij} = 1$ for every $i$, and $a_{ij} ge 0$. Let
$$begin{equation}
begin{pmatrix}
y_1 \
vdots \
y_n \
end{pmatrix}
=mathbf{A}
begin{pmatrix}
x_1 \
vdots \
x_n
end{pmatrix}
end{equation}$$

with non-negative $y_i$ and $x_i$. Prove that $y_1 cdots y_n ge x_1 cdots x_n$.



It may somehow matter to convex function.










share|cite|improve this question









New contributor




X.T Chen is a new contributor to this site. Take care in asking for clarification, commenting, and answering.
Check out our Code of Conduct.
















  • 1




    I suppose that with your inequalities you mean that $y$ majorizes $x$?
    – lcv
    2 days ago






  • 1




    Posted also on Mathematics: Question about an inequation described by matrices. In my opinion, this answer gives a very reasonable advice on cross-posting.
    – Martin Sleziak
    yesterday
















4














Let $A=(a_{ij})_{1 le i, j le n}$ be a matrix such that $sum_limits{i=1}^{n} a_{ij}=1$ for every $j$, and $sum_limits{j=1}^n a_{ij} = 1$ for every $i$, and $a_{ij} ge 0$. Let
$$begin{equation}
begin{pmatrix}
y_1 \
vdots \
y_n \
end{pmatrix}
=mathbf{A}
begin{pmatrix}
x_1 \
vdots \
x_n
end{pmatrix}
end{equation}$$

with non-negative $y_i$ and $x_i$. Prove that $y_1 cdots y_n ge x_1 cdots x_n$.



It may somehow matter to convex function.










share|cite|improve this question









New contributor




X.T Chen is a new contributor to this site. Take care in asking for clarification, commenting, and answering.
Check out our Code of Conduct.
















  • 1




    I suppose that with your inequalities you mean that $y$ majorizes $x$?
    – lcv
    2 days ago






  • 1




    Posted also on Mathematics: Question about an inequation described by matrices. In my opinion, this answer gives a very reasonable advice on cross-posting.
    – Martin Sleziak
    yesterday














4












4








4


4





Let $A=(a_{ij})_{1 le i, j le n}$ be a matrix such that $sum_limits{i=1}^{n} a_{ij}=1$ for every $j$, and $sum_limits{j=1}^n a_{ij} = 1$ for every $i$, and $a_{ij} ge 0$. Let
$$begin{equation}
begin{pmatrix}
y_1 \
vdots \
y_n \
end{pmatrix}
=mathbf{A}
begin{pmatrix}
x_1 \
vdots \
x_n
end{pmatrix}
end{equation}$$

with non-negative $y_i$ and $x_i$. Prove that $y_1 cdots y_n ge x_1 cdots x_n$.



It may somehow matter to convex function.










share|cite|improve this question









New contributor




X.T Chen is a new contributor to this site. Take care in asking for clarification, commenting, and answering.
Check out our Code of Conduct.











Let $A=(a_{ij})_{1 le i, j le n}$ be a matrix such that $sum_limits{i=1}^{n} a_{ij}=1$ for every $j$, and $sum_limits{j=1}^n a_{ij} = 1$ for every $i$, and $a_{ij} ge 0$. Let
$$begin{equation}
begin{pmatrix}
y_1 \
vdots \
y_n \
end{pmatrix}
=mathbf{A}
begin{pmatrix}
x_1 \
vdots \
x_n
end{pmatrix}
end{equation}$$

with non-negative $y_i$ and $x_i$. Prove that $y_1 cdots y_n ge x_1 cdots x_n$.



It may somehow matter to convex function.







linear-algebra inequalities convex-analysis






share|cite|improve this question









New contributor




X.T Chen is a new contributor to this site. Take care in asking for clarification, commenting, and answering.
Check out our Code of Conduct.











share|cite|improve this question









New contributor




X.T Chen is a new contributor to this site. Take care in asking for clarification, commenting, and answering.
Check out our Code of Conduct.









share|cite|improve this question




share|cite|improve this question








edited yesterday









Alexandre Eremenko

49k6136253




49k6136253






New contributor




X.T Chen is a new contributor to this site. Take care in asking for clarification, commenting, and answering.
Check out our Code of Conduct.









asked 2 days ago









X.T Chen

212




212




New contributor




X.T Chen is a new contributor to this site. Take care in asking for clarification, commenting, and answering.
Check out our Code of Conduct.





New contributor





X.T Chen is a new contributor to this site. Take care in asking for clarification, commenting, and answering.
Check out our Code of Conduct.






X.T Chen is a new contributor to this site. Take care in asking for clarification, commenting, and answering.
Check out our Code of Conduct.








  • 1




    I suppose that with your inequalities you mean that $y$ majorizes $x$?
    – lcv
    2 days ago






  • 1




    Posted also on Mathematics: Question about an inequation described by matrices. In my opinion, this answer gives a very reasonable advice on cross-posting.
    – Martin Sleziak
    yesterday














  • 1




    I suppose that with your inequalities you mean that $y$ majorizes $x$?
    – lcv
    2 days ago






  • 1




    Posted also on Mathematics: Question about an inequation described by matrices. In my opinion, this answer gives a very reasonable advice on cross-posting.
    – Martin Sleziak
    yesterday








1




1




I suppose that with your inequalities you mean that $y$ majorizes $x$?
– lcv
2 days ago




I suppose that with your inequalities you mean that $y$ majorizes $x$?
– lcv
2 days ago




1




1




Posted also on Mathematics: Question about an inequation described by matrices. In my opinion, this answer gives a very reasonable advice on cross-posting.
– Martin Sleziak
yesterday




Posted also on Mathematics: Question about an inequation described by matrices. In my opinion, this answer gives a very reasonable advice on cross-posting.
– Martin Sleziak
yesterday










3 Answers
3






active

oldest

votes


















5














$$y_i=sum_j a_{ij} x_jgeqslant prod_j x_j^{a_{ij} }$$
by Jensen inequality for logarithm. Now take the product over $i=1,2,dots,n$.






share|cite|improve this answer































    3














    This is a special case of the so called Schur's majorization inequality. Here are the details.
    $newcommand{bR}{mathbb{R}}$ $newcommand{bx}{boldsymbol{x}}$ $newcommand{by}{boldsymbol{y}}$



    Given $bxinbR^n$ we denote by $bar{bx}$ the vector obtained from $bx$ by rearranging its coordinates in decreasing order. We say that $bx$ dominates $by$ and we write this $bxsuccby$ if



    $$sum_{i=1}^k bar{x}_igeq sum_{i=1}^k bar{y}_i,;;forall k=1,dotsc, n-1, $$



    $$sum_{i=1}^n bar{x}_i= sum_{i=1}^n bar{y}_i.$$



    The symmetric group $S_n$ acts on $bR^n$ by permuting the coordinates of a vector. For $bxinbR^n$ we denote by $S_ncdotbx$ the orbit of $bx$ with respect to this action, i.e., the set of vectors that can be obtained from $bx$ by permuting its coordinates. $DeclareMathOperator{conv}{conv}$



    For a set $Ssubset bR^n$ we denote by $conv(S)$ its convex hull.



    The next nontrivial theorem characterizes the dominance relation. For a nice presentation of this theorem and Schur's majorization inequality I refer to Chapter 13 of




    J.Michael Steele: The Cauchy-Schwarz Master Class, Cambridge University Press, 2004.




    Theorem. Let $bx,byinbR^n$. The following statements are equivalent.





    1. $bxsucc by$.


    2. $byin conv( S_ncdotbx)$.

    3. There exists a doubly stochastic $ntimes n$ matrix $A$ such that $by=Abx$.


    Fix an interval $(a,b)subset bR$. A symmetric $C^1$-function $f:(a,b)^ntobR$ is called Schur convex if $newcommand{pa}{partial}$



    $$ (x_j-x_k)left( frac{pa f}{pa x_j}(bx)-frac{pa f}{pa x_k}(bx)right)geq 0,$$



    for any $bxin (a,b)^n$ and any $j,k=1,dotsc, n$.



    The Schur majorization inequality states that



    $$ bxsucc by implies f(bx)geq f(by), $$



    for any Schur convex function $f:(a,b)^ntobR$ and any $bx,byin(a,b)^n$.



    The function



    $$ p:[0,infty)^ntobR,;;p(bx)=-x_1cdots x_n $$



    is Schur convex. The inequality $p(bx)geq p(by)$ is the inequality that interests you.






    share|cite|improve this answer































      2














      By the equivalence of conditions (ii) and (iv) in Theorem A.3 on page 14, the condition $y=Ax$ for $y=[y,dots,y_n]^T$ and $x=[x,dots,x_n]^T$ is equivalent to the condition that $sum_1^n g(y_i)gesum_1^n g(x_i)$ for all continuous concave functions $g$. Now take $g=ln$ to get the desired inequality $y_1 cdots y_n ge x_1 cdots x_n$.






      share|cite|improve this answer





















        Your Answer





        StackExchange.ifUsing("editor", function () {
        return StackExchange.using("mathjaxEditing", function () {
        StackExchange.MarkdownEditor.creationCallbacks.add(function (editor, postfix) {
        StackExchange.mathjaxEditing.prepareWmdForMathJax(editor, postfix, [["$", "$"], ["\\(","\\)"]]);
        });
        });
        }, "mathjax-editing");

        StackExchange.ready(function() {
        var channelOptions = {
        tags: "".split(" "),
        id: "504"
        };
        initTagRenderer("".split(" "), "".split(" "), channelOptions);

        StackExchange.using("externalEditor", function() {
        // Have to fire editor after snippets, if snippets enabled
        if (StackExchange.settings.snippets.snippetsEnabled) {
        StackExchange.using("snippets", function() {
        createEditor();
        });
        }
        else {
        createEditor();
        }
        });

        function createEditor() {
        StackExchange.prepareEditor({
        heartbeatType: 'answer',
        autoActivateHeartbeat: false,
        convertImagesToLinks: true,
        noModals: true,
        showLowRepImageUploadWarning: true,
        reputationToPostImages: 10,
        bindNavPrevention: true,
        postfix: "",
        imageUploader: {
        brandingHtml: "Powered by u003ca class="icon-imgur-white" href="https://imgur.com/"u003eu003c/au003e",
        contentPolicyHtml: "User contributions licensed under u003ca href="https://creativecommons.org/licenses/by-sa/3.0/"u003ecc by-sa 3.0 with attribution requiredu003c/au003e u003ca href="https://stackoverflow.com/legal/content-policy"u003e(content policy)u003c/au003e",
        allowUrls: true
        },
        noCode: true, onDemand: true,
        discardSelector: ".discard-answer"
        ,immediatelyShowMarkdownHelp:true
        });


        }
        });






        X.T Chen is a new contributor. Be nice, and check out our Code of Conduct.










        draft saved

        draft discarded


















        StackExchange.ready(
        function () {
        StackExchange.openid.initPostLogin('.new-post-login', 'https%3a%2f%2fmathoverflow.net%2fquestions%2f319474%2fquestion-about-an-inequality-described-by-matrices%23new-answer', 'question_page');
        }
        );

        Post as a guest















        Required, but never shown

























        3 Answers
        3






        active

        oldest

        votes








        3 Answers
        3






        active

        oldest

        votes









        active

        oldest

        votes






        active

        oldest

        votes









        5














        $$y_i=sum_j a_{ij} x_jgeqslant prod_j x_j^{a_{ij} }$$
        by Jensen inequality for logarithm. Now take the product over $i=1,2,dots,n$.






        share|cite|improve this answer




























          5














          $$y_i=sum_j a_{ij} x_jgeqslant prod_j x_j^{a_{ij} }$$
          by Jensen inequality for logarithm. Now take the product over $i=1,2,dots,n$.






          share|cite|improve this answer


























            5












            5








            5






            $$y_i=sum_j a_{ij} x_jgeqslant prod_j x_j^{a_{ij} }$$
            by Jensen inequality for logarithm. Now take the product over $i=1,2,dots,n$.






            share|cite|improve this answer














            $$y_i=sum_j a_{ij} x_jgeqslant prod_j x_j^{a_{ij} }$$
            by Jensen inequality for logarithm. Now take the product over $i=1,2,dots,n$.







            share|cite|improve this answer














            share|cite|improve this answer



            share|cite|improve this answer








            edited yesterday









            Todd Trimble

            43.4k5156257




            43.4k5156257










            answered yesterday









            Fedor Petrov

            47.3k5111218




            47.3k5111218























                3














                This is a special case of the so called Schur's majorization inequality. Here are the details.
                $newcommand{bR}{mathbb{R}}$ $newcommand{bx}{boldsymbol{x}}$ $newcommand{by}{boldsymbol{y}}$



                Given $bxinbR^n$ we denote by $bar{bx}$ the vector obtained from $bx$ by rearranging its coordinates in decreasing order. We say that $bx$ dominates $by$ and we write this $bxsuccby$ if



                $$sum_{i=1}^k bar{x}_igeq sum_{i=1}^k bar{y}_i,;;forall k=1,dotsc, n-1, $$



                $$sum_{i=1}^n bar{x}_i= sum_{i=1}^n bar{y}_i.$$



                The symmetric group $S_n$ acts on $bR^n$ by permuting the coordinates of a vector. For $bxinbR^n$ we denote by $S_ncdotbx$ the orbit of $bx$ with respect to this action, i.e., the set of vectors that can be obtained from $bx$ by permuting its coordinates. $DeclareMathOperator{conv}{conv}$



                For a set $Ssubset bR^n$ we denote by $conv(S)$ its convex hull.



                The next nontrivial theorem characterizes the dominance relation. For a nice presentation of this theorem and Schur's majorization inequality I refer to Chapter 13 of




                J.Michael Steele: The Cauchy-Schwarz Master Class, Cambridge University Press, 2004.




                Theorem. Let $bx,byinbR^n$. The following statements are equivalent.





                1. $bxsucc by$.


                2. $byin conv( S_ncdotbx)$.

                3. There exists a doubly stochastic $ntimes n$ matrix $A$ such that $by=Abx$.


                Fix an interval $(a,b)subset bR$. A symmetric $C^1$-function $f:(a,b)^ntobR$ is called Schur convex if $newcommand{pa}{partial}$



                $$ (x_j-x_k)left( frac{pa f}{pa x_j}(bx)-frac{pa f}{pa x_k}(bx)right)geq 0,$$



                for any $bxin (a,b)^n$ and any $j,k=1,dotsc, n$.



                The Schur majorization inequality states that



                $$ bxsucc by implies f(bx)geq f(by), $$



                for any Schur convex function $f:(a,b)^ntobR$ and any $bx,byin(a,b)^n$.



                The function



                $$ p:[0,infty)^ntobR,;;p(bx)=-x_1cdots x_n $$



                is Schur convex. The inequality $p(bx)geq p(by)$ is the inequality that interests you.






                share|cite|improve this answer




























                  3














                  This is a special case of the so called Schur's majorization inequality. Here are the details.
                  $newcommand{bR}{mathbb{R}}$ $newcommand{bx}{boldsymbol{x}}$ $newcommand{by}{boldsymbol{y}}$



                  Given $bxinbR^n$ we denote by $bar{bx}$ the vector obtained from $bx$ by rearranging its coordinates in decreasing order. We say that $bx$ dominates $by$ and we write this $bxsuccby$ if



                  $$sum_{i=1}^k bar{x}_igeq sum_{i=1}^k bar{y}_i,;;forall k=1,dotsc, n-1, $$



                  $$sum_{i=1}^n bar{x}_i= sum_{i=1}^n bar{y}_i.$$



                  The symmetric group $S_n$ acts on $bR^n$ by permuting the coordinates of a vector. For $bxinbR^n$ we denote by $S_ncdotbx$ the orbit of $bx$ with respect to this action, i.e., the set of vectors that can be obtained from $bx$ by permuting its coordinates. $DeclareMathOperator{conv}{conv}$



                  For a set $Ssubset bR^n$ we denote by $conv(S)$ its convex hull.



                  The next nontrivial theorem characterizes the dominance relation. For a nice presentation of this theorem and Schur's majorization inequality I refer to Chapter 13 of




                  J.Michael Steele: The Cauchy-Schwarz Master Class, Cambridge University Press, 2004.




                  Theorem. Let $bx,byinbR^n$. The following statements are equivalent.





                  1. $bxsucc by$.


                  2. $byin conv( S_ncdotbx)$.

                  3. There exists a doubly stochastic $ntimes n$ matrix $A$ such that $by=Abx$.


                  Fix an interval $(a,b)subset bR$. A symmetric $C^1$-function $f:(a,b)^ntobR$ is called Schur convex if $newcommand{pa}{partial}$



                  $$ (x_j-x_k)left( frac{pa f}{pa x_j}(bx)-frac{pa f}{pa x_k}(bx)right)geq 0,$$



                  for any $bxin (a,b)^n$ and any $j,k=1,dotsc, n$.



                  The Schur majorization inequality states that



                  $$ bxsucc by implies f(bx)geq f(by), $$



                  for any Schur convex function $f:(a,b)^ntobR$ and any $bx,byin(a,b)^n$.



                  The function



                  $$ p:[0,infty)^ntobR,;;p(bx)=-x_1cdots x_n $$



                  is Schur convex. The inequality $p(bx)geq p(by)$ is the inequality that interests you.






                  share|cite|improve this answer


























                    3












                    3








                    3






                    This is a special case of the so called Schur's majorization inequality. Here are the details.
                    $newcommand{bR}{mathbb{R}}$ $newcommand{bx}{boldsymbol{x}}$ $newcommand{by}{boldsymbol{y}}$



                    Given $bxinbR^n$ we denote by $bar{bx}$ the vector obtained from $bx$ by rearranging its coordinates in decreasing order. We say that $bx$ dominates $by$ and we write this $bxsuccby$ if



                    $$sum_{i=1}^k bar{x}_igeq sum_{i=1}^k bar{y}_i,;;forall k=1,dotsc, n-1, $$



                    $$sum_{i=1}^n bar{x}_i= sum_{i=1}^n bar{y}_i.$$



                    The symmetric group $S_n$ acts on $bR^n$ by permuting the coordinates of a vector. For $bxinbR^n$ we denote by $S_ncdotbx$ the orbit of $bx$ with respect to this action, i.e., the set of vectors that can be obtained from $bx$ by permuting its coordinates. $DeclareMathOperator{conv}{conv}$



                    For a set $Ssubset bR^n$ we denote by $conv(S)$ its convex hull.



                    The next nontrivial theorem characterizes the dominance relation. For a nice presentation of this theorem and Schur's majorization inequality I refer to Chapter 13 of




                    J.Michael Steele: The Cauchy-Schwarz Master Class, Cambridge University Press, 2004.




                    Theorem. Let $bx,byinbR^n$. The following statements are equivalent.





                    1. $bxsucc by$.


                    2. $byin conv( S_ncdotbx)$.

                    3. There exists a doubly stochastic $ntimes n$ matrix $A$ such that $by=Abx$.


                    Fix an interval $(a,b)subset bR$. A symmetric $C^1$-function $f:(a,b)^ntobR$ is called Schur convex if $newcommand{pa}{partial}$



                    $$ (x_j-x_k)left( frac{pa f}{pa x_j}(bx)-frac{pa f}{pa x_k}(bx)right)geq 0,$$



                    for any $bxin (a,b)^n$ and any $j,k=1,dotsc, n$.



                    The Schur majorization inequality states that



                    $$ bxsucc by implies f(bx)geq f(by), $$



                    for any Schur convex function $f:(a,b)^ntobR$ and any $bx,byin(a,b)^n$.



                    The function



                    $$ p:[0,infty)^ntobR,;;p(bx)=-x_1cdots x_n $$



                    is Schur convex. The inequality $p(bx)geq p(by)$ is the inequality that interests you.






                    share|cite|improve this answer














                    This is a special case of the so called Schur's majorization inequality. Here are the details.
                    $newcommand{bR}{mathbb{R}}$ $newcommand{bx}{boldsymbol{x}}$ $newcommand{by}{boldsymbol{y}}$



                    Given $bxinbR^n$ we denote by $bar{bx}$ the vector obtained from $bx$ by rearranging its coordinates in decreasing order. We say that $bx$ dominates $by$ and we write this $bxsuccby$ if



                    $$sum_{i=1}^k bar{x}_igeq sum_{i=1}^k bar{y}_i,;;forall k=1,dotsc, n-1, $$



                    $$sum_{i=1}^n bar{x}_i= sum_{i=1}^n bar{y}_i.$$



                    The symmetric group $S_n$ acts on $bR^n$ by permuting the coordinates of a vector. For $bxinbR^n$ we denote by $S_ncdotbx$ the orbit of $bx$ with respect to this action, i.e., the set of vectors that can be obtained from $bx$ by permuting its coordinates. $DeclareMathOperator{conv}{conv}$



                    For a set $Ssubset bR^n$ we denote by $conv(S)$ its convex hull.



                    The next nontrivial theorem characterizes the dominance relation. For a nice presentation of this theorem and Schur's majorization inequality I refer to Chapter 13 of




                    J.Michael Steele: The Cauchy-Schwarz Master Class, Cambridge University Press, 2004.




                    Theorem. Let $bx,byinbR^n$. The following statements are equivalent.





                    1. $bxsucc by$.


                    2. $byin conv( S_ncdotbx)$.

                    3. There exists a doubly stochastic $ntimes n$ matrix $A$ such that $by=Abx$.


                    Fix an interval $(a,b)subset bR$. A symmetric $C^1$-function $f:(a,b)^ntobR$ is called Schur convex if $newcommand{pa}{partial}$



                    $$ (x_j-x_k)left( frac{pa f}{pa x_j}(bx)-frac{pa f}{pa x_k}(bx)right)geq 0,$$



                    for any $bxin (a,b)^n$ and any $j,k=1,dotsc, n$.



                    The Schur majorization inequality states that



                    $$ bxsucc by implies f(bx)geq f(by), $$



                    for any Schur convex function $f:(a,b)^ntobR$ and any $bx,byin(a,b)^n$.



                    The function



                    $$ p:[0,infty)^ntobR,;;p(bx)=-x_1cdots x_n $$



                    is Schur convex. The inequality $p(bx)geq p(by)$ is the inequality that interests you.







                    share|cite|improve this answer














                    share|cite|improve this answer



                    share|cite|improve this answer








                    edited yesterday

























                    answered yesterday









                    Liviu Nicolaescu

                    25.4k258110




                    25.4k258110























                        2














                        By the equivalence of conditions (ii) and (iv) in Theorem A.3 on page 14, the condition $y=Ax$ for $y=[y,dots,y_n]^T$ and $x=[x,dots,x_n]^T$ is equivalent to the condition that $sum_1^n g(y_i)gesum_1^n g(x_i)$ for all continuous concave functions $g$. Now take $g=ln$ to get the desired inequality $y_1 cdots y_n ge x_1 cdots x_n$.






                        share|cite|improve this answer


























                          2














                          By the equivalence of conditions (ii) and (iv) in Theorem A.3 on page 14, the condition $y=Ax$ for $y=[y,dots,y_n]^T$ and $x=[x,dots,x_n]^T$ is equivalent to the condition that $sum_1^n g(y_i)gesum_1^n g(x_i)$ for all continuous concave functions $g$. Now take $g=ln$ to get the desired inequality $y_1 cdots y_n ge x_1 cdots x_n$.






                          share|cite|improve this answer
























                            2












                            2








                            2






                            By the equivalence of conditions (ii) and (iv) in Theorem A.3 on page 14, the condition $y=Ax$ for $y=[y,dots,y_n]^T$ and $x=[x,dots,x_n]^T$ is equivalent to the condition that $sum_1^n g(y_i)gesum_1^n g(x_i)$ for all continuous concave functions $g$. Now take $g=ln$ to get the desired inequality $y_1 cdots y_n ge x_1 cdots x_n$.






                            share|cite|improve this answer












                            By the equivalence of conditions (ii) and (iv) in Theorem A.3 on page 14, the condition $y=Ax$ for $y=[y,dots,y_n]^T$ and $x=[x,dots,x_n]^T$ is equivalent to the condition that $sum_1^n g(y_i)gesum_1^n g(x_i)$ for all continuous concave functions $g$. Now take $g=ln$ to get the desired inequality $y_1 cdots y_n ge x_1 cdots x_n$.







                            share|cite|improve this answer












                            share|cite|improve this answer



                            share|cite|improve this answer










                            answered 2 days ago









                            Iosif Pinelis

                            17.7k12158




                            17.7k12158






















                                X.T Chen is a new contributor. Be nice, and check out our Code of Conduct.










                                draft saved

                                draft discarded


















                                X.T Chen is a new contributor. Be nice, and check out our Code of Conduct.













                                X.T Chen is a new contributor. Be nice, and check out our Code of Conduct.












                                X.T Chen is a new contributor. Be nice, and check out our Code of Conduct.
















                                Thanks for contributing an answer to MathOverflow!


                                • Please be sure to answer the question. Provide details and share your research!

                                But avoid



                                • Asking for help, clarification, or responding to other answers.

                                • Making statements based on opinion; back them up with references or personal experience.


                                Use MathJax to format equations. MathJax reference.


                                To learn more, see our tips on writing great answers.





                                Some of your past answers have not been well-received, and you're in danger of being blocked from answering.


                                Please pay close attention to the following guidance:


                                • Please be sure to answer the question. Provide details and share your research!

                                But avoid



                                • Asking for help, clarification, or responding to other answers.

                                • Making statements based on opinion; back them up with references or personal experience.


                                To learn more, see our tips on writing great answers.




                                draft saved


                                draft discarded














                                StackExchange.ready(
                                function () {
                                StackExchange.openid.initPostLogin('.new-post-login', 'https%3a%2f%2fmathoverflow.net%2fquestions%2f319474%2fquestion-about-an-inequality-described-by-matrices%23new-answer', 'question_page');
                                }
                                );

                                Post as a guest















                                Required, but never shown





















































                                Required, but never shown














                                Required, but never shown












                                Required, but never shown







                                Required, but never shown

































                                Required, but never shown














                                Required, but never shown












                                Required, but never shown







                                Required, but never shown







                                Popular posts from this blog

                                If I really need a card on my start hand, how many mulligans make sense? [duplicate]

                                Alcedinidae

                                Can an atomic nucleus contain both particles and antiparticles? [duplicate]